$fcirc fcirc f(x)=x^9$ then $f$ is increasing











up vote
5
down vote

favorite
1












The full statement is:



If $f:Bbb R to Bbb R$ is a continuous function and $fcirc fcirc f(x)=x^9$ then $f$ is increasing.



$(1)$ I was thinking about suppose that $f$ is decreasing (or constant) and then it is easy to get a contradiction.



After that I would like to state that



"If $f:Bbb R to Bbb R$ is continous and not increasing then there is an interval where $f$ is decreasing or constant."



and then I can use $(1)$ and get the result. But I still can't prove if the statement is true.



Any hint or any other solution?










share|cite|improve this question


















  • 4




    $f$ is injective. A continuous injective function is either increasing or decreasing (I am not sure to have understood if this was what you were asking for though)
    – Charles Madeline
    Jul 20 '17 at 13:29








  • 1




    @charMD: That helps a lot! Thanks!
    – Arnaldo
    Jul 20 '17 at 13:34

















up vote
5
down vote

favorite
1












The full statement is:



If $f:Bbb R to Bbb R$ is a continuous function and $fcirc fcirc f(x)=x^9$ then $f$ is increasing.



$(1)$ I was thinking about suppose that $f$ is decreasing (or constant) and then it is easy to get a contradiction.



After that I would like to state that



"If $f:Bbb R to Bbb R$ is continous and not increasing then there is an interval where $f$ is decreasing or constant."



and then I can use $(1)$ and get the result. But I still can't prove if the statement is true.



Any hint or any other solution?










share|cite|improve this question


















  • 4




    $f$ is injective. A continuous injective function is either increasing or decreasing (I am not sure to have understood if this was what you were asking for though)
    – Charles Madeline
    Jul 20 '17 at 13:29








  • 1




    @charMD: That helps a lot! Thanks!
    – Arnaldo
    Jul 20 '17 at 13:34















up vote
5
down vote

favorite
1









up vote
5
down vote

favorite
1






1





The full statement is:



If $f:Bbb R to Bbb R$ is a continuous function and $fcirc fcirc f(x)=x^9$ then $f$ is increasing.



$(1)$ I was thinking about suppose that $f$ is decreasing (or constant) and then it is easy to get a contradiction.



After that I would like to state that



"If $f:Bbb R to Bbb R$ is continous and not increasing then there is an interval where $f$ is decreasing or constant."



and then I can use $(1)$ and get the result. But I still can't prove if the statement is true.



Any hint or any other solution?










share|cite|improve this question













The full statement is:



If $f:Bbb R to Bbb R$ is a continuous function and $fcirc fcirc f(x)=x^9$ then $f$ is increasing.



$(1)$ I was thinking about suppose that $f$ is decreasing (or constant) and then it is easy to get a contradiction.



After that I would like to state that



"If $f:Bbb R to Bbb R$ is continous and not increasing then there is an interval where $f$ is decreasing or constant."



and then I can use $(1)$ and get the result. But I still can't prove if the statement is true.



Any hint or any other solution?







real-analysis functions continuity






share|cite|improve this question













share|cite|improve this question











share|cite|improve this question




share|cite|improve this question










asked Jul 20 '17 at 13:18









Arnaldo

18.1k42246




18.1k42246








  • 4




    $f$ is injective. A continuous injective function is either increasing or decreasing (I am not sure to have understood if this was what you were asking for though)
    – Charles Madeline
    Jul 20 '17 at 13:29








  • 1




    @charMD: That helps a lot! Thanks!
    – Arnaldo
    Jul 20 '17 at 13:34
















  • 4




    $f$ is injective. A continuous injective function is either increasing or decreasing (I am not sure to have understood if this was what you were asking for though)
    – Charles Madeline
    Jul 20 '17 at 13:29








  • 1




    @charMD: That helps a lot! Thanks!
    – Arnaldo
    Jul 20 '17 at 13:34










4




4




$f$ is injective. A continuous injective function is either increasing or decreasing (I am not sure to have understood if this was what you were asking for though)
– Charles Madeline
Jul 20 '17 at 13:29






$f$ is injective. A continuous injective function is either increasing or decreasing (I am not sure to have understood if this was what you were asking for though)
– Charles Madeline
Jul 20 '17 at 13:29






1




1




@charMD: That helps a lot! Thanks!
– Arnaldo
Jul 20 '17 at 13:34






@charMD: That helps a lot! Thanks!
– Arnaldo
Jul 20 '17 at 13:34












2 Answers
2






active

oldest

votes

















up vote
1
down vote













I think that your statement is true.



Let $a,b$ be points, $b>a$ so that $f(b) leq f(a)$. Let $c$ be the $x$ value where $f$ attains its maximum on the interval $[a,b]$. Clearly $c neq b$ (or at least we can take $c neq b$), and hence for some $epsilon> 0$, $f$ will be decreasing or constant on the interval: $(c, c + epsilon)$.






share|cite|improve this answer





















  • Is this true? The Weierstrass function is not increasing or decreasing on any interval.
    – Solomonoff's Secret
    Jul 20 '17 at 13:56


















up vote
0
down vote













Since $x mapsto x^9$ is one-to-one and onto, the function $f$ itself must also be one-to-one and onto. Together with continuity this means that $f$ is strictly monotone, so it must be strictly increasing.



As remarked in comments to some other answers, general continuous functions may fail to be monotone on any interval.






share|cite|improve this answer





















    Your Answer





    StackExchange.ifUsing("editor", function () {
    return StackExchange.using("mathjaxEditing", function () {
    StackExchange.MarkdownEditor.creationCallbacks.add(function (editor, postfix) {
    StackExchange.mathjaxEditing.prepareWmdForMathJax(editor, postfix, [["$", "$"], ["\\(","\\)"]]);
    });
    });
    }, "mathjax-editing");

    StackExchange.ready(function() {
    var channelOptions = {
    tags: "".split(" "),
    id: "69"
    };
    initTagRenderer("".split(" "), "".split(" "), channelOptions);

    StackExchange.using("externalEditor", function() {
    // Have to fire editor after snippets, if snippets enabled
    if (StackExchange.settings.snippets.snippetsEnabled) {
    StackExchange.using("snippets", function() {
    createEditor();
    });
    }
    else {
    createEditor();
    }
    });

    function createEditor() {
    StackExchange.prepareEditor({
    heartbeatType: 'answer',
    convertImagesToLinks: true,
    noModals: true,
    showLowRepImageUploadWarning: true,
    reputationToPostImages: 10,
    bindNavPrevention: true,
    postfix: "",
    imageUploader: {
    brandingHtml: "Powered by u003ca class="icon-imgur-white" href="https://imgur.com/"u003eu003c/au003e",
    contentPolicyHtml: "User contributions licensed under u003ca href="https://creativecommons.org/licenses/by-sa/3.0/"u003ecc by-sa 3.0 with attribution requiredu003c/au003e u003ca href="https://stackoverflow.com/legal/content-policy"u003e(content policy)u003c/au003e",
    allowUrls: true
    },
    noCode: true, onDemand: true,
    discardSelector: ".discard-answer"
    ,immediatelyShowMarkdownHelp:true
    });


    }
    });














    draft saved

    draft discarded


















    StackExchange.ready(
    function () {
    StackExchange.openid.initPostLogin('.new-post-login', 'https%3a%2f%2fmath.stackexchange.com%2fquestions%2f2365006%2ff-circ-f-circ-fx-x9-then-f-is-increasing%23new-answer', 'question_page');
    }
    );

    Post as a guest















    Required, but never shown

























    2 Answers
    2






    active

    oldest

    votes








    2 Answers
    2






    active

    oldest

    votes









    active

    oldest

    votes






    active

    oldest

    votes








    up vote
    1
    down vote













    I think that your statement is true.



    Let $a,b$ be points, $b>a$ so that $f(b) leq f(a)$. Let $c$ be the $x$ value where $f$ attains its maximum on the interval $[a,b]$. Clearly $c neq b$ (or at least we can take $c neq b$), and hence for some $epsilon> 0$, $f$ will be decreasing or constant on the interval: $(c, c + epsilon)$.






    share|cite|improve this answer





















    • Is this true? The Weierstrass function is not increasing or decreasing on any interval.
      – Solomonoff's Secret
      Jul 20 '17 at 13:56















    up vote
    1
    down vote













    I think that your statement is true.



    Let $a,b$ be points, $b>a$ so that $f(b) leq f(a)$. Let $c$ be the $x$ value where $f$ attains its maximum on the interval $[a,b]$. Clearly $c neq b$ (or at least we can take $c neq b$), and hence for some $epsilon> 0$, $f$ will be decreasing or constant on the interval: $(c, c + epsilon)$.






    share|cite|improve this answer





















    • Is this true? The Weierstrass function is not increasing or decreasing on any interval.
      – Solomonoff's Secret
      Jul 20 '17 at 13:56













    up vote
    1
    down vote










    up vote
    1
    down vote









    I think that your statement is true.



    Let $a,b$ be points, $b>a$ so that $f(b) leq f(a)$. Let $c$ be the $x$ value where $f$ attains its maximum on the interval $[a,b]$. Clearly $c neq b$ (or at least we can take $c neq b$), and hence for some $epsilon> 0$, $f$ will be decreasing or constant on the interval: $(c, c + epsilon)$.






    share|cite|improve this answer












    I think that your statement is true.



    Let $a,b$ be points, $b>a$ so that $f(b) leq f(a)$. Let $c$ be the $x$ value where $f$ attains its maximum on the interval $[a,b]$. Clearly $c neq b$ (or at least we can take $c neq b$), and hence for some $epsilon> 0$, $f$ will be decreasing or constant on the interval: $(c, c + epsilon)$.







    share|cite|improve this answer












    share|cite|improve this answer



    share|cite|improve this answer










    answered Jul 20 '17 at 13:39









    Mohit

    960513




    960513












    • Is this true? The Weierstrass function is not increasing or decreasing on any interval.
      – Solomonoff's Secret
      Jul 20 '17 at 13:56


















    • Is this true? The Weierstrass function is not increasing or decreasing on any interval.
      – Solomonoff's Secret
      Jul 20 '17 at 13:56
















    Is this true? The Weierstrass function is not increasing or decreasing on any interval.
    – Solomonoff's Secret
    Jul 20 '17 at 13:56




    Is this true? The Weierstrass function is not increasing or decreasing on any interval.
    – Solomonoff's Secret
    Jul 20 '17 at 13:56










    up vote
    0
    down vote













    Since $x mapsto x^9$ is one-to-one and onto, the function $f$ itself must also be one-to-one and onto. Together with continuity this means that $f$ is strictly monotone, so it must be strictly increasing.



    As remarked in comments to some other answers, general continuous functions may fail to be monotone on any interval.






    share|cite|improve this answer

























      up vote
      0
      down vote













      Since $x mapsto x^9$ is one-to-one and onto, the function $f$ itself must also be one-to-one and onto. Together with continuity this means that $f$ is strictly monotone, so it must be strictly increasing.



      As remarked in comments to some other answers, general continuous functions may fail to be monotone on any interval.






      share|cite|improve this answer























        up vote
        0
        down vote










        up vote
        0
        down vote









        Since $x mapsto x^9$ is one-to-one and onto, the function $f$ itself must also be one-to-one and onto. Together with continuity this means that $f$ is strictly monotone, so it must be strictly increasing.



        As remarked in comments to some other answers, general continuous functions may fail to be monotone on any interval.






        share|cite|improve this answer












        Since $x mapsto x^9$ is one-to-one and onto, the function $f$ itself must also be one-to-one and onto. Together with continuity this means that $f$ is strictly monotone, so it must be strictly increasing.



        As remarked in comments to some other answers, general continuous functions may fail to be monotone on any interval.







        share|cite|improve this answer












        share|cite|improve this answer



        share|cite|improve this answer










        answered Nov 20 at 21:01









        Lukas Geyer

        13.2k1454




        13.2k1454






























            draft saved

            draft discarded




















































            Thanks for contributing an answer to Mathematics Stack Exchange!


            • Please be sure to answer the question. Provide details and share your research!

            But avoid



            • Asking for help, clarification, or responding to other answers.

            • Making statements based on opinion; back them up with references or personal experience.


            Use MathJax to format equations. MathJax reference.


            To learn more, see our tips on writing great answers.





            Some of your past answers have not been well-received, and you're in danger of being blocked from answering.


            Please pay close attention to the following guidance:


            • Please be sure to answer the question. Provide details and share your research!

            But avoid



            • Asking for help, clarification, or responding to other answers.

            • Making statements based on opinion; back them up with references or personal experience.


            To learn more, see our tips on writing great answers.




            draft saved


            draft discarded














            StackExchange.ready(
            function () {
            StackExchange.openid.initPostLogin('.new-post-login', 'https%3a%2f%2fmath.stackexchange.com%2fquestions%2f2365006%2ff-circ-f-circ-fx-x9-then-f-is-increasing%23new-answer', 'question_page');
            }
            );

            Post as a guest















            Required, but never shown





















































            Required, but never shown














            Required, but never shown












            Required, but never shown







            Required, but never shown

































            Required, but never shown














            Required, but never shown












            Required, but never shown







            Required, but never shown







            Popular posts from this blog

            Plaza Victoria

            In PowerPoint, is there a keyboard shortcut for bulleted / numbered list?

            How to put 3 figures in Latex with 2 figures side by side and 1 below these side by side images but in...